2
$\begingroup$

There are multiple results on the sieve method, and I wanted to ask about the following variant (to know if it is trivial by one of the current versions of the sieve method, or seems a challenging problem)

Let $\varepsilon>0$ be fixed. Then there exists $0<\delta<\varepsilon$, such that for all sufficiently large $k$ the following is true.

Let the primes strictly between $\sqrt k$ and $k$ be written as $\sqrt k <p_1\le \ldots \le p_r <k$. Let $M= \prod_{j=1}^r p_j$. For every $1 \le j \le r$, let $I_j= \left \{1,2,\ldots, \left \lceil p_j^{1-\delta} \right \rceil \right \}$. Then among every $M^\varepsilon$ consecutive integers, there is at least one value $x$ for which $ x \bmod p_j$ belongs to $I_j$ for every $1 \le j \le r$.

Note: By the prime number theorem, $M=k^{\Theta( k/ \log{k} )}$. The average size of intersection is at least $M^{\varepsilon-\delta}$, which is large for $k$ large. One can e.g. assume $(\varepsilon-\delta) k^{0.01}> \log{k}$. So maybe this is easy for the experts.

Thanks in advance.

ps: if the problem is solved, with almost all k instead of all k, I would also be happy. But then the proof may use different ideas.

$\endgroup$
7
  • $\begingroup$ what does "choose intervals $I_j$ with the first $p_j^{1-\delta}$ numbers" mean? $\endgroup$ Commented Sep 22, 2024 at 18:26
  • $\begingroup$ You need to define $I_j$ and $M$ more explicitly. The way it's currently written the question is hard to parse. $\endgroup$ Commented Sep 22, 2024 at 19:11
  • 1
    $\begingroup$ @AlexeiEntin I believe that $M=\prod_{j=1}^r p_j$, so probably $M$ is the product of the primes in $[\sqrt{k},k]$. Also, I believe that $I_j=\mathbb{N}\cap[1,p_j^{1-\delta}]$. $\endgroup$ Commented Sep 22, 2024 at 19:15
  • $\begingroup$ The above (reply by GH) is correct. Should have written first $p_j^{1-\delta}$ natural numbers to be clearer. Alternatively, one can also aim more general; for every $I_j\subset [p_j]$ for which $\vert I_j \rvert = p_j^{1-\delta}$. $\endgroup$ Commented Sep 22, 2024 at 20:01
  • 1
    $\begingroup$ Question has been edited to clarify all the issues. $\endgroup$ Commented Sep 23, 2024 at 5:53

1 Answer 1

0
$\begingroup$

As an update; it is now part of https://arxiv.org/pdf/2410.09138

$\endgroup$
1
  • $\begingroup$ It says there, "For example when $x$ is even and $y > x$ is odd and not much larger than $x$, we may have that ${\rm lcm}(x, x + 1, x + 2) = x(x + 1)(x + 2) >y(y+1)(y+2)/2 = {\rm lcm}(y, y + 1, y + 2)$." But when $x$ is even and $y$ is odd, then ${\rm lcm}(x, x + 1, x + 2) = x(x + 1)(x + 2)/2$ and $y(y+1)(y+2) = {\rm lcm}(y, y + 1, y + 2)$ $\endgroup$ Commented Jul 28 at 7:53

You must log in to answer this question.

Start asking to get answers

Find the answer to your question by asking.

Ask question

Explore related questions

See similar questions with these tags.